Discuss the convergence of the series $sum_{n=2}^infty(ln{n})^{-ln(ln{n})}$












1












$begingroup$


Discuss the convergence or divergence of the series where the general term $x_n$ is given by



$$x_n=(ln{n})^{-ln(ln{n})},,,nge 2.$$










share|cite|improve this question











$endgroup$












  • $begingroup$
    Cauchy's Condensation Test gives you a clearly divergent series (its general term doesn't even converge to zero...)
    $endgroup$
    – DonAntonio
    Mar 16 '14 at 13:20










  • $begingroup$
    I have not learnt Cauchy Condensation Test. Can you tell me how it works? Thanks
    $endgroup$
    – anonymous
    Mar 16 '14 at 13:26










  • $begingroup$
    Google it, @anonymous....:)
    $endgroup$
    – DonAntonio
    Mar 16 '14 at 13:27










  • $begingroup$
    @DonAntonio: Are you saying $x_n not to 0$ as $n to infty$?
    $endgroup$
    – Henry
    Mar 16 '14 at 13:29






  • 2




    $begingroup$
    @Henry No, he's saying $2^kcdot x_{2^k} notto 0$. (Or maybe replace the $2$ with a different base, the condensation test isn't tied to $2$.)
    $endgroup$
    – Daniel Fischer
    Mar 16 '14 at 13:32
















1












$begingroup$


Discuss the convergence or divergence of the series where the general term $x_n$ is given by



$$x_n=(ln{n})^{-ln(ln{n})},,,nge 2.$$










share|cite|improve this question











$endgroup$












  • $begingroup$
    Cauchy's Condensation Test gives you a clearly divergent series (its general term doesn't even converge to zero...)
    $endgroup$
    – DonAntonio
    Mar 16 '14 at 13:20










  • $begingroup$
    I have not learnt Cauchy Condensation Test. Can you tell me how it works? Thanks
    $endgroup$
    – anonymous
    Mar 16 '14 at 13:26










  • $begingroup$
    Google it, @anonymous....:)
    $endgroup$
    – DonAntonio
    Mar 16 '14 at 13:27










  • $begingroup$
    @DonAntonio: Are you saying $x_n not to 0$ as $n to infty$?
    $endgroup$
    – Henry
    Mar 16 '14 at 13:29






  • 2




    $begingroup$
    @Henry No, he's saying $2^kcdot x_{2^k} notto 0$. (Or maybe replace the $2$ with a different base, the condensation test isn't tied to $2$.)
    $endgroup$
    – Daniel Fischer
    Mar 16 '14 at 13:32














1












1








1


1



$begingroup$


Discuss the convergence or divergence of the series where the general term $x_n$ is given by



$$x_n=(ln{n})^{-ln(ln{n})},,,nge 2.$$










share|cite|improve this question











$endgroup$




Discuss the convergence or divergence of the series where the general term $x_n$ is given by



$$x_n=(ln{n})^{-ln(ln{n})},,,nge 2.$$







calculus real-analysis sequences-and-series limits convergence






share|cite|improve this question















share|cite|improve this question













share|cite|improve this question




share|cite|improve this question








edited Aug 23 '14 at 14:55









Yiorgos S. Smyrlis

62.9k1384163




62.9k1384163










asked Mar 16 '14 at 13:15









anonymousanonymous

211




211












  • $begingroup$
    Cauchy's Condensation Test gives you a clearly divergent series (its general term doesn't even converge to zero...)
    $endgroup$
    – DonAntonio
    Mar 16 '14 at 13:20










  • $begingroup$
    I have not learnt Cauchy Condensation Test. Can you tell me how it works? Thanks
    $endgroup$
    – anonymous
    Mar 16 '14 at 13:26










  • $begingroup$
    Google it, @anonymous....:)
    $endgroup$
    – DonAntonio
    Mar 16 '14 at 13:27










  • $begingroup$
    @DonAntonio: Are you saying $x_n not to 0$ as $n to infty$?
    $endgroup$
    – Henry
    Mar 16 '14 at 13:29






  • 2




    $begingroup$
    @Henry No, he's saying $2^kcdot x_{2^k} notto 0$. (Or maybe replace the $2$ with a different base, the condensation test isn't tied to $2$.)
    $endgroup$
    – Daniel Fischer
    Mar 16 '14 at 13:32


















  • $begingroup$
    Cauchy's Condensation Test gives you a clearly divergent series (its general term doesn't even converge to zero...)
    $endgroup$
    – DonAntonio
    Mar 16 '14 at 13:20










  • $begingroup$
    I have not learnt Cauchy Condensation Test. Can you tell me how it works? Thanks
    $endgroup$
    – anonymous
    Mar 16 '14 at 13:26










  • $begingroup$
    Google it, @anonymous....:)
    $endgroup$
    – DonAntonio
    Mar 16 '14 at 13:27










  • $begingroup$
    @DonAntonio: Are you saying $x_n not to 0$ as $n to infty$?
    $endgroup$
    – Henry
    Mar 16 '14 at 13:29






  • 2




    $begingroup$
    @Henry No, he's saying $2^kcdot x_{2^k} notto 0$. (Or maybe replace the $2$ with a different base, the condensation test isn't tied to $2$.)
    $endgroup$
    – Daniel Fischer
    Mar 16 '14 at 13:32
















$begingroup$
Cauchy's Condensation Test gives you a clearly divergent series (its general term doesn't even converge to zero...)
$endgroup$
– DonAntonio
Mar 16 '14 at 13:20




$begingroup$
Cauchy's Condensation Test gives you a clearly divergent series (its general term doesn't even converge to zero...)
$endgroup$
– DonAntonio
Mar 16 '14 at 13:20












$begingroup$
I have not learnt Cauchy Condensation Test. Can you tell me how it works? Thanks
$endgroup$
– anonymous
Mar 16 '14 at 13:26




$begingroup$
I have not learnt Cauchy Condensation Test. Can you tell me how it works? Thanks
$endgroup$
– anonymous
Mar 16 '14 at 13:26












$begingroup$
Google it, @anonymous....:)
$endgroup$
– DonAntonio
Mar 16 '14 at 13:27




$begingroup$
Google it, @anonymous....:)
$endgroup$
– DonAntonio
Mar 16 '14 at 13:27












$begingroup$
@DonAntonio: Are you saying $x_n not to 0$ as $n to infty$?
$endgroup$
– Henry
Mar 16 '14 at 13:29




$begingroup$
@DonAntonio: Are you saying $x_n not to 0$ as $n to infty$?
$endgroup$
– Henry
Mar 16 '14 at 13:29




2




2




$begingroup$
@Henry No, he's saying $2^kcdot x_{2^k} notto 0$. (Or maybe replace the $2$ with a different base, the condensation test isn't tied to $2$.)
$endgroup$
– Daniel Fischer
Mar 16 '14 at 13:32




$begingroup$
@Henry No, he's saying $2^kcdot x_{2^k} notto 0$. (Or maybe replace the $2$ with a different base, the condensation test isn't tied to $2$.)
$endgroup$
– Daniel Fischer
Mar 16 '14 at 13:32










1 Answer
1






active

oldest

votes


















10












$begingroup$

We have that
$$
(ln n)^{-ln ln n}=mathrm{e}^{-(ln ln n)^2}>mathrm{e}^{-ln n}=frac{1}{n},
$$
for $n$ sufficiently large since
$$
lim_{ntoinfty}frac{(ln ln n)^2}{ln n}=lim_{Ntoinfty}frac{(ln N)^2}{N}=0.
$$
Hence the series
$$
sum_{n=2}^infty(ln n)^{-ln ln n}
$$
diverges to infinity, by virtue of the comparison test.






share|cite|improve this answer









$endgroup$













    Your Answer





    StackExchange.ifUsing("editor", function () {
    return StackExchange.using("mathjaxEditing", function () {
    StackExchange.MarkdownEditor.creationCallbacks.add(function (editor, postfix) {
    StackExchange.mathjaxEditing.prepareWmdForMathJax(editor, postfix, [["$", "$"], ["\\(","\\)"]]);
    });
    });
    }, "mathjax-editing");

    StackExchange.ready(function() {
    var channelOptions = {
    tags: "".split(" "),
    id: "69"
    };
    initTagRenderer("".split(" "), "".split(" "), channelOptions);

    StackExchange.using("externalEditor", function() {
    // Have to fire editor after snippets, if snippets enabled
    if (StackExchange.settings.snippets.snippetsEnabled) {
    StackExchange.using("snippets", function() {
    createEditor();
    });
    }
    else {
    createEditor();
    }
    });

    function createEditor() {
    StackExchange.prepareEditor({
    heartbeatType: 'answer',
    autoActivateHeartbeat: false,
    convertImagesToLinks: true,
    noModals: true,
    showLowRepImageUploadWarning: true,
    reputationToPostImages: 10,
    bindNavPrevention: true,
    postfix: "",
    imageUploader: {
    brandingHtml: "Powered by u003ca class="icon-imgur-white" href="https://imgur.com/"u003eu003c/au003e",
    contentPolicyHtml: "User contributions licensed under u003ca href="https://creativecommons.org/licenses/by-sa/3.0/"u003ecc by-sa 3.0 with attribution requiredu003c/au003e u003ca href="https://stackoverflow.com/legal/content-policy"u003e(content policy)u003c/au003e",
    allowUrls: true
    },
    noCode: true, onDemand: true,
    discardSelector: ".discard-answer"
    ,immediatelyShowMarkdownHelp:true
    });


    }
    });














    draft saved

    draft discarded


















    StackExchange.ready(
    function () {
    StackExchange.openid.initPostLogin('.new-post-login', 'https%3a%2f%2fmath.stackexchange.com%2fquestions%2f714227%2fdiscuss-the-convergence-of-the-series-sum-n-2-infty-lnn-ln-lnn%23new-answer', 'question_page');
    }
    );

    Post as a guest















    Required, but never shown

























    1 Answer
    1






    active

    oldest

    votes








    1 Answer
    1






    active

    oldest

    votes









    active

    oldest

    votes






    active

    oldest

    votes









    10












    $begingroup$

    We have that
    $$
    (ln n)^{-ln ln n}=mathrm{e}^{-(ln ln n)^2}>mathrm{e}^{-ln n}=frac{1}{n},
    $$
    for $n$ sufficiently large since
    $$
    lim_{ntoinfty}frac{(ln ln n)^2}{ln n}=lim_{Ntoinfty}frac{(ln N)^2}{N}=0.
    $$
    Hence the series
    $$
    sum_{n=2}^infty(ln n)^{-ln ln n}
    $$
    diverges to infinity, by virtue of the comparison test.






    share|cite|improve this answer









    $endgroup$


















      10












      $begingroup$

      We have that
      $$
      (ln n)^{-ln ln n}=mathrm{e}^{-(ln ln n)^2}>mathrm{e}^{-ln n}=frac{1}{n},
      $$
      for $n$ sufficiently large since
      $$
      lim_{ntoinfty}frac{(ln ln n)^2}{ln n}=lim_{Ntoinfty}frac{(ln N)^2}{N}=0.
      $$
      Hence the series
      $$
      sum_{n=2}^infty(ln n)^{-ln ln n}
      $$
      diverges to infinity, by virtue of the comparison test.






      share|cite|improve this answer









      $endgroup$
















        10












        10








        10





        $begingroup$

        We have that
        $$
        (ln n)^{-ln ln n}=mathrm{e}^{-(ln ln n)^2}>mathrm{e}^{-ln n}=frac{1}{n},
        $$
        for $n$ sufficiently large since
        $$
        lim_{ntoinfty}frac{(ln ln n)^2}{ln n}=lim_{Ntoinfty}frac{(ln N)^2}{N}=0.
        $$
        Hence the series
        $$
        sum_{n=2}^infty(ln n)^{-ln ln n}
        $$
        diverges to infinity, by virtue of the comparison test.






        share|cite|improve this answer









        $endgroup$



        We have that
        $$
        (ln n)^{-ln ln n}=mathrm{e}^{-(ln ln n)^2}>mathrm{e}^{-ln n}=frac{1}{n},
        $$
        for $n$ sufficiently large since
        $$
        lim_{ntoinfty}frac{(ln ln n)^2}{ln n}=lim_{Ntoinfty}frac{(ln N)^2}{N}=0.
        $$
        Hence the series
        $$
        sum_{n=2}^infty(ln n)^{-ln ln n}
        $$
        diverges to infinity, by virtue of the comparison test.







        share|cite|improve this answer












        share|cite|improve this answer



        share|cite|improve this answer










        answered Mar 16 '14 at 13:29









        Yiorgos S. SmyrlisYiorgos S. Smyrlis

        62.9k1384163




        62.9k1384163






























            draft saved

            draft discarded




















































            Thanks for contributing an answer to Mathematics Stack Exchange!


            • Please be sure to answer the question. Provide details and share your research!

            But avoid



            • Asking for help, clarification, or responding to other answers.

            • Making statements based on opinion; back them up with references or personal experience.


            Use MathJax to format equations. MathJax reference.


            To learn more, see our tips on writing great answers.




            draft saved


            draft discarded














            StackExchange.ready(
            function () {
            StackExchange.openid.initPostLogin('.new-post-login', 'https%3a%2f%2fmath.stackexchange.com%2fquestions%2f714227%2fdiscuss-the-convergence-of-the-series-sum-n-2-infty-lnn-ln-lnn%23new-answer', 'question_page');
            }
            );

            Post as a guest















            Required, but never shown





















































            Required, but never shown














            Required, but never shown












            Required, but never shown







            Required, but never shown

































            Required, but never shown














            Required, but never shown












            Required, but never shown







            Required, but never shown







            Popular posts from this blog

            Quarter-circle Tiles

            build a pushdown automaton that recognizes the reverse language of a given pushdown automaton?

            Mont Emei